Hat Peskin & Schroeder Gl. (4.26), U(t1,t2)U(t2,t3)=U(t1,t3)U(t1,t2)U(t2,t3)=U(t1,t3)U(t_1,t_2)U( t_2,t_3) = U(t_1,t_3) impliziert [H0,Hinweis]=0[H0,Hinweis]=0[H_0,H_{int}] = 0?

Peskin & Schroeder-Gleichung (4.17) definieren den Operator,

(4.17) U ( T , T 0 )   =   e ich ( T T 0 ) H 0 e ich ( T T 0 ) H
Wo
(4.12) H   =   H 0 + H int
ist der vollständige Hamiltonoperator und H 0 ist der freie Hamilton, beide im Schrödinger-Bild. In Gleichung (4.26) geben Peskin und Schroeder an, dass der Operator die folgende Identität erfüllt,
(4.26) U ( T 1 , T 2 ) U ( T 2 , T 3 )   =   U ( T 1 , T 3 )
Wo T 1 T 2 T 3 . Bedeutet dies, dass der freie Hamiltonoperator mit der Wechselwirkung pendelt?
[ H 0 , H int ]   =   0   ?
Hier ist mein Argument, dass dies der Fall ist.

Im Zustand T 1 T 2 T 3 nehmen T 2 = 0 . Die Identität ist dann

U ( T 1 , 0 ) U ( 0 , T 3 ) = U ( T 1 , T 3 )   .
Ersetzen Sie die Definition,
e ich T 1 H 0 e ich T 1 H e ich T 3 H 0 e ich T 3 H = e ich ( T 1 T 3 ) H 0 e ich ( T 1 T 3 ) H
und vereinfachen zu bekommen,
e ich T 1 H e ich T 3 H 0 = e ich T 3 H 0 e ich T 1 H
mit T 1 0 T 3 . Setzen T 1 = T Und T 3 = T .
e ich T H e ich T H 0 = e ich T H 0 e ich T H
Erweiterung auf zweite Ordnung in T ,
( 1 ich T H T 2 2 H H ) ( 1 + ich T H 0 T 2 2 H 0 H 0 ) = ( 1 + ich T H 0 T 2 2 H 0 H 0 ) ( 1 ich T H T 2 2 H H )
ergibt,
H H 0 = H 0 H
so dass [ H 0 , H ] = 0 . Jetzt H = H 0 + H ich N T also muss der freie Hamiltonoperator mit der Wechselwirkung pendeln.
[ H 0 , H ich N T ] = 0
Bei Peskin und Schroeder ist der Kontext für dieses Material das selbstinteragierende Skalarfeld mit Hamiltonian,
H = D 3 X ( 1 2 π ( T , X ) 2 + 1 2 ϕ X R ϕ X R + v ( ϕ ) )   .
In der klassischen Theorie ist der PB
[ H 0 , H ich N T ] P B = D 3 X δ H 0 δ π δ H ich N T δ ϕ = D 3 X   π D v D ϕ = D D T D 3 X   v ( ϕ ( T , X ) )
Gehen wir zur Quantentheorie über,
[ H 0 , H ich N T ] = ich D D T D 3 X   v ( ϕ ( T , X ) )
so dass [ H 0 , H ich N T ] = 0   impliziert das Integral von v ( ϕ ) eine Erhaltungsladung ist; Ist das auch ein korrektes Ergebnis?

Es gibt keine solche Anforderung an H 0 Und H ich N T . Es muss ein Tippfehler sein --- die korrekte Definition des Evolutionsoperators im Interaktionsbild ist e ich H T konjugiert von e ich H 0 T .
Ich glaube es ist ein Tippfehler.
Dies scheitert bereits mit einfachem Masselos ϕ 4 Theorie. Deutlich, ϕ 4 kann nicht mit pendeln Π ϕ , und wird daher nicht mit dem Hamiltonoperator kommutieren.
"Impliziert dies, dass der freie Hamiltonoperator mit der Wechselwirkung [H0,Hint]=0 pendelt?" Nein, tut es nicht.

Antworten (3)

Ref. 1 schreibt die richtige Formel

(4.25) U ( T , T ' )   =   e ich H 0 ( T T 0 ) e ich H ( T T ' ) e ich H 0 ( T ' T 0 ) , T     T ' ,

was befriedigt

(4.26) U ( T 1 , T 2 ) U ( T 2 , T 3 )   =   U ( T 1 , T 3 ) , T 1     T 2     T 3 .

Hier T 0 ist ein willkürlicher, aber fester Referenz-Anfangszeitpunkt, an dem Operatoren und Zustände im Schrödinger-Bild , im Heisenberg-Bild und im Wechselwirkungsbild übereinstimmen. Für T T 0 , sind die drei Bilder nicht mehr dieselben, obwohl sie immer noch einheitliches Äquivalent sind.

Für T ' = T 0 , Gl. (4.25) vereinfacht sich zu

(4.17) U ( T , T 0 )   =   e ich H 0 ( T T 0 ) e ich H ( T T 0 ) .

Es scheint, dass OP fälschlicherweise ersetzt T 0 in Gl. (4.17) mit beliebiger Zeit T ' T . Die resultierende Gleichung

U ( T , T ' )   =   e ich H 0 ( T T ' ) e ich H ( T T ' ) . ( Falsch! )

ist nicht richtig.

Verweise:

  1. ME Peskin & DV Schroeder, Eine Einführung in QFT; Abschnitt 4.2.

Die Tatsache, dass

(1) U ICH ( T 1 , T 2 ) U ICH ( T 2 , T 3 ) = U ICH ( T 1 , T 3 )
im Interaktionsbild nicht darauf angewiesen H 0 Und H int Pendeln, kann aber ohne diese Annahme aus der Tomonaga-Schwinger-Gleichung abgeleitet werden
ich T U ICH ( T , T 0 ) = H ICH ( T ) U ICH ( T , T 0 )
mit H ICH ( T ) := e ich H 0 T H int e ich H 0 T als H int entwickelt von H 0 und dass diese Gleichung eine Dyson-Reihenlösung hat
U ICH ( T , T 0 ) = T exp ( ich T 0 T H ICH ( T ' ) D T ' )
aus denen ( 1 ) kann angezeigt werden.

Die Fassung von U ICH von Peskin-Schröder (und einigen anderen, die vermutlich davon kopiert haben) falsch ist, ist die richtige Version

U ICH ( T , T 0 ) = e ich H 0 T e ich H ( T T 0 ) e ich H 0 T 0
was folgt aus ψ ICH ( T ) := e ich H 0 T ψ S ( T ) Und ψ ICH ( T ) = U ICH ( T , T 0 ) ψ ICH ( T 0 ) , Wo S bezeichnet Schrödinger-Zustände. Dies erfüllt offensichtlich auch ( 1 ) ohne zusätzliche Annahmen.

Zuerst haben Sie Taylors Erweiterungen zweiter Ordnung genommen e ich T H Und e ich T H 0 , also, wenn Ihre folgenden Berechnungen richtig wären, H Und H 0 nur in zweiter Ordnung miteinander pendeln würden T , nicht für alle Bestellungen T (oder nur ungefähr pendeln).

Zweitens für 4 Operatoren A , B , H , K :

A H B = A K B A ( H K ) B = 0.
Aus dieser Beziehung können wir ableiten H = K wenn (und nur wenn) für alle A, B (H, K bleiben unverändert), gilt die obige Beziehung immer. In Ihren Berechnungen gilt
A = e ich T 1 H 0 , B = e ich T 3 H , H = e ich T 1 H e ich T 3 H 0 , K = e ich T 3 H 0 e ich T 1 H
Wir sehen das, A Und K hier sind miteinander verwandt (auch z B Und H ). Aus der obigen Bedingung können wir also nicht schließen H = K .

Dein Ergebnis stimmt also nicht.

A Und B Umkehrungen haben. Also ab A H B = A K B , A 1 A H B B 1 = A 1 A K B B 1 impliziert H = K .
Außerdem gilt für Ihren ersten Punkt die Gleichheit der Potenzreihen für alle T , und es gibt keine anderen Terme zweiter Ordnung aus den Erweiterungen höherer Ordnung. Da für zwei konvergente Reihen N A N T N = N B N T N T dann und nur dann, wenn A N = B N , wäre das Argument im OP korrekt (es verwendet A 2 = B 2 ), wenn die angegebene Form von U ICH waren richtig.